weiwu0221 Wrote:(A) is the best answer available, but it looks like a sufficient condition rather than a neccessary one for the modifier "only". Because it preludes all other possibilities, it's more than neccessary. What do you think about that?
~(patent rights) →
~(financial incentive) +
~(parent rights)
→
~(engagement in original development / new inventions)Just because something is necessary doesn't mean that it is NOT sufficient also. The question of "necessary or sufficient?," in true LSAT fashion, does not preclude the possibility of both. I would argue that (A)
is a sufficient condition. If we say that ~(financial incentive) → ~(motivation to develop new inventions) then the argument seems to follow logically with no holes.
However, it is also necessary. If financial reward is NOT the ONLY incentive, then that means there are other incentives. Could the conclusion still be true? Oh yea! Does the conclusion follow from the premises? Absolutely not. Why? Because sticking in this negated answer choice (A) would make you think, "why did you even bring up the premises then, LSAT writer?!" This is a subtle indication that the conclusion simply doesn't follow logically from the premises.
djy_neworiental Wrote:I have the same question. I understand A can fill the gap but it seems more like a sufficent assumption because of the word "only". So if I negate A and financial reward is not the only incentive, the conc seems to be ok. Because, you know, what if financial reward is the primary incentive?
and I chose c because when I negate c, I ask myself what if the cost is more significant than the future profit? the conc seems to fall apart.
(1) see above. Necessary and sufficient are not exclusive terms.
(2) The conclusion may seem to be "okay," as in it could still be true, but the point is that the conclusion wouldn't FOLLOW from the PREMISES. That is what a necessary assumption is all about. If you negate something and the conclusion seems to not make all that much sense, you are doing it right!
(3) When you arrived at (C) you seemed to be committing the very same flaw this question is testing you on. You seemed to have assumed that "financial incentive" was critical to the development of inventions. It may not be.
csunnerberg13 Wrote:Why is A not incorrect as a sufficient assumption??
See above!
Hope that's helpful!